A sprinter run 400 meter in 54 second.what about s the runner's average running rate in meter per second?round to the nearest tenth

Answers

Answer 1

Answer:

7.4

Step-by-step explanation:

400 ÷ 54 =7.407407...

7.407407... rounded to the nearest tenth is 7.4

I hope this helps... and plz mark me brainliest!!!


Related Questions

Evaluate the function below at x=5. Then, enter your solution. f(x)=3(2)^x

Answers

Answer:

Solution: f(5) = 96

Step-by-step explanation:

f(5) = 3(2)^5

f(5) = 3 (2 × 2 × 2 × 2 × 2)

f(5) = 3 (32)

f(5) = 96

Which of the following is true? Tangent is positive in Quadrant I. Sine is negative in Quadrant II. Cosine is positive in Quadrant III. Sine is positive in Quadrant IV.

Answers

A) Tangent is positive in Quadrant I.

Since sine and cosine are both positive in Quadrant I and tangent is the ratio of sine to cosine, tangent is positive in Quadrant I

Answer:

A

Step-by-step explanation:

I had this question and got it right the user above explains it in detail

PLEASE HELP, WILL GIVE BRAINLIEST IF CORRECT!!!! (08.06 MC) Mike and his friends bought cheese wafers for $2 per packet and chocolate wafers for $1 per packet at a carnival. They spent a total of $25 to buy a total of 20 packets of wafers of the two varieties. Part A: Write a system of equations that can be solved to find the number of packets of cheese wafers and the number of packets of chocolate wafers that Mike and his friends bought at the carnival. Define the variables used in the equations. (5 points) Part B: How many packets of chocolate wafers and cheese wafers did they buy? Explain how you got the answer and why you selected a particular method to get the answer. (5 points)

Answers

Answer:

x = 5 , y = 15

Step-by-step explanation:

You can solve this using substitution.

Let the quantity of cheese wafers be denoted by x and the quantity of chocolate wafers denoted by y

2x + 1y = 25

x + y = 20

These two equations are the answer to part A, (remember to include the above prompt which says what x and y denote).

For part B I used substitution because it was more applicable to the question then addition or elimination.

ACTUAL WORK

Set 2x + 1y = 25 equal to x

x = 25 - y / 2

Replace x with y in the second equation

(25 - y / 2) + y = 20

And solve for y

y = 15

Since we know what y is we can replace y in the second equation and find what x is

x + 15 = 20

Solve for x

x = 5

Answer:

5 Cheese Wafers and 15 Chocolate Wafers

Step-by-step explanation:

A bag contains twelve marbles, which includes seven red marbles and five blue marbles. Roja reaches into the bag and pulls out four marbles. a) How many different sets of four marbles can be pulled from this bag? b) How many of these sets contain two red marbles and two blue marbles? c) How many of these sets contain all red marbles? d) How many of these sets contain all red marbles or all blue marbles?

Answers

Answer:

a) 495

b) 210

c) 35

d) 40

Step-by-step explanation:

Given a total of 12 marbles.

n = 12

Number of red marbles = 7

Number of blue marbles = 5

a) Number of different sets of 4 marbles that can be made from this bag ?

This is a simple combination problem.

where n = 12 and r = 4.

So, answer will be:

[tex]_{12}C_4[/tex]

Formula:

[tex]_{n}C_r = \dfrac{n!}{(n-r)!r!}[/tex]

[tex]_{12}C_4 = \dfrac{12!}{(8)!4!} = \dfrac{12\times 11\times 10\times 9}{4 \times 3\times 2} =\bold{495}[/tex]

b) Two red and two blue marbles:

The answer will be:

[tex]_{7}C_2 \times _{5}C_2 = \dfrac{7\times 6}{2} \times \dfrac{5\times 4}{2} =\bold{210}[/tex]

c) all red marbles.(4 chosen out of 7 red and 0 chosen out of 5 blue marbles)

[tex]_{7}C_4 \times _{5}C_0 = \dfrac{7\times 6\times 5\times 4}{4\times 3\times 2} =\bold{35}[/tex]

d) all red or all blue.(all red marbles plus all blue marbles)

All red marbles:

[tex]_{7}C_4 \times _{5}C_0 = \dfrac{7\times 6\times 5\times 4}{4\times 3\times 2} \times 1=\bold{35}[/tex]

All blue marbles:

[tex]_{7}C_0 \times _{5}C_4 = 1 \times \dfrac{5\times 4\times 3\times 2}{4\times 3\times 2} =\bold{5}[/tex]

So, answer is 40.

A combination lock uses three numbers between 1 and 46 with​ repetition, and they must be selected in the correct sequence. Is the name of​ "combination lock"​ appropriate? Why or why​ not? Choose the correct answer below. A. ​No, because the multiplication counting rule would be used to determine the total number of combinations. B. ​Yes, because the combinations rule would be used to determine the total number of combinations. C. ​No, because factorials would be used to determine the total number of combinations. D. ​No, because the permutations rule would be used to determine the total number of combinations.

Answers

The correct answer is D. ​No because the permutations rule would be used to determine the total number of combinations.

Explanation:

The difference between a combination and a permutation is that in permutations the order is considered. This applies to the numbers in a lock because these need to be in order. Therefore, to analyze the permutations in a lock, the rule for permutations should be used. This includes the general formula P (n,r) =[tex]\frac{n!}{(n-r) !}[/tex]; in this, n is the number of objects and r refers to the objects used in a permutation. Thus, the term "combination" is inappropriate because this is a permutation, and the permutation rule should be used.

Pls help, I don’t know how to fo

Answers

frustum of a cone is: = pi * l(R + r)

(l) = slant height of the frustum.

from 2929.645714 - 506.1257143

= 2423.52

= 2423.5cm

Answer:

from 2929.645714 - 506.1257143

= 2423.52

= 2423.5cm

Latanya buys 5 yard of blue fabric and 8 yards of green fabric. the blue fabric cost $2 dollars more than the green fabric.she pays a total of $ 62. what would be the combined cost of 1 yard of blue fabric and one yard of green fabric?

Answers

Answer: $10

Step-by-step explanation:

let x = the price of green fabric, then x+2 = blue fabric price

8x+5(x+2)=62

8x+5x+10=62

    13x+10=62

          13x=52

              x=4

price of green fabric=$4

price of blue fabric=$6

4+6=$10

Solve. 4x−y−2z=−8 −2x+4z=−4 x+2y=6 Enter your answer, in the form (x,y,z), in the boxes in simplest terms. x= y= z=

Answers

Answer:

(-2, 4, 2)

Where x = -2, y = 4, and z = 2.

Step-by-step explanation:

We are given the system of three equations:

[tex]\displaystyle \left\{ \begin{array}{l} 4x -y -2z = -8 \\ -2x + 4z = -4 \\ x + 2y = 6 \end{array}[/tex]

And we want to find the value of each variable.

Note that both the second and third equations have an x.

Therefore, we can isolate the variables for the second and third equation and then substitute them into the first equation to make the first equation all one variable.

Solve the second equation for z:

[tex]\displaystyle \begin{aligned} -2x+4z&=-4 \\ x - 2 &= 2z \\ z&= \frac{x-2}{2}\end{aligned}[/tex]

Likewise, solve the third equation for y:

[tex]\displaystyle \begin{aligned} x+2y &= 6\\ 2y &= 6-x \\ y &= \frac{6-x}{2} \end{aligned}[/tex]

Substitute the above equations into the first:

[tex]\displaystyle 4x - \left(\frac{6-x}{2}\right) - 2\left(\frac{x-2}{2}\right)=-8[/tex]

And solve for x:

[tex]\displaystyle \begin{aligned} 4x+\left(\frac{x-6}{2}\right)+(2-x) &= -8 \\ \\ 8x +(x-6) +(4-2x) &= -16 \\ \\ 7x-2 &= -16 \\ \\ 7x &= -14 \\ \\ x &= -2\end{aligned}[/tex]

Hence, x = -2.

Find z and y using their respective equations:

Second equation:

[tex]\displaystyle \begin{aligned} z&=\frac{x-2}{2} \\ &= \frac{(-2)-2}{2} \\ &= \frac{-4}{2} \\ &= -2\end{aligned}[/tex]

Third equation:

[tex]\displaystyle \begin{aligned} y &= \frac{6-x}{2}\\ &= \frac{6-(-2)}{2}\\ &= \frac{8}{2}\\ &=4\end{aligned}[/tex]

In conclusion, the solution is (-2, 4, -2)

Answer:

x = -2

y =4

z=-2

Step-by-step explanation:

4x−y−2z=−8

−2x+4z=−4

x+2y=6

Solve the second equation for x

x = 6 -2y

Substitute into the first two equations

4x−y−2z=−8

4(6-2y) -y -2 = 8  

24 -8y-y -2z = 8

-9y -2z = -32

−2(6-2y)+4z=−4

-12 +4y +4z = -4

4y+4z = 8

Divide by 4

y+z = 2

z =2-y

Substitute this into -9y -2z = -32

-9y -2(2-y) = -32

-9y -4 +2y = -32

-7y -4 = -32

-7y =-28

y =4

Now find z

z = 2-y

z = 2-4

z = -2

Now find x

x = 6 -2y

x = 6 -2(4)

x =6-8

x = -2

Can someone help me with this please it’s algebra 2

Answers

Answer:

7 8 9

Step-by-step explanation:

If 4SINB=3SIN(2A+B) :
Prove that:7COT(A+B)=COTA

Answers

Answer:

Step-by-step explanation:

Given the expression 4sinB = 3sin(2A+B), we are to show that the expression 7cot(A+B) = cotA

Starting with the expression

4sinB= 3sin(2A+B)

Let us re write angle B = (A + B) - A

and 2A + B = (A + B) + A

Substituting the derived expression back into the original expression ww will have;

4Sin{(A + B) - A } = 3Sin{(A + B)+ A}

From trigonometry identity;

Sin(D+E) = SinDcosE + CosDSinE

Sin(D-E) = SinDcosE - CosDSinE

Applying this in the expression above;

4{Sin(A+B)CosA - Cos(A+B)SinA} = 3{Sin(A+B)CosA + Cos(A+B)sinA}

Open the bracket

4Sin(A+B)CosA - 4Cos(A+B)SinA = 3Sin(A+B)CosA + 3Cos(A+B)sinA

Collecting like terms

4Sin(A+B)CosA - 3Sin(A+B)cosA = 3Cos(A+B)sinA + 4Cos(A+B)sinA

Sin(A+B)CosA = 7Cos(A+B)sinA

Divide both sides by sinA

Sin(A+B)CosA/sinA= 7Cos(A+B)sinA/sinA

Since cosA/sinA = cotA, the expression becomes;

Sin(A+B)cotA = 7Cos(A+B)

Finally, divide both sides of the resulting equation by sin(A+B)

Sin(A+B)cotA/sin(A+B) = 7Cos(A+B)/sin(A+B)

CotA = 7cot(A+B) Proved!

PLEASE HELP QUICK, WILL MARK BRAINLIEST!
Solve for x: −6 < x − 1 < 9

5 < x < 10
−5 < x < 10
−5 > x > 10
5 > x > −10

Answers

Answer:

−5 < x < 10

Step-by-step explanation:

−6 < x − 1 < 9

Add 1 to all sides

−6+1 < x − 1+1 < 9+1

−5 < x < 10

Answer:

B

Step-by-step explanation:

Add one to everything

-5 < x < 10

Best of Luck!

how many are 6 raised to 4 ???​

Answers

Answer:

[tex]\large \boxed{1296}[/tex]

Step-by-step explanation:

6 raised to 4 indicates that the base 6 has an exponent or power of 4.

[tex]6^4[/tex]

6 is multiplied by itself 4 times.

[tex]6 \times 6 \times 6 \times 6[/tex]

[tex]=1296[/tex]

determine the image of the point p[-3,10) under the translation [5,-7]

Answers

[tex](-3+5,10-7)=(2,3)[/tex]

solve this equation -2x+9=-5x-15

Answers

Answer:

x = -8

I hope this helps!

The answer would be x=-8

please can someone help me solve this.. please help!!​

Answers

Step-by-step explanation:

Hello,

Firstly just look to triangle BDE,

Here, you will find that,

140° = y+80° {the exterior and opposite interior angle of a triangle is equal}.

or, y= 140°-80° {shifting 80° to another side and subtracting it.}

Therefore, the value of y is 60°.

now, let's simply work with line EB or EG. we get;

angle GEF + y=180° { being a linear pair}.

or, angle GEF + 60°= 180°

or, angle GEF = 180°-60°

Therefore, the value of angle GEF = 120°.

now, looking in triangle EFG, we get;

angle GEF + 35°+x= 180° { the sum of interior angle of a triangle is 180°}.

or, 120°+35°+ x= 180°

or, x= 180°- 155°

Therefore, the value of x is 25°.

now, lastly finding the value of "z"

We find that x= z {being vertical opposite angle}

or, z =25°

Therefore, the value of z is 25°.

So, the values are,

x=25°

y=60°

and z= 25°

Hope it helps...

simpily 2^3×3^2=6^5​

Answers

Answer:

2^3×3^2=6^5​  equation is wrong because

2×2×2×3×3=72

6^5=6×6×6×6×6=36×36×6=7776

the two numbers are not equal

Mate, I think your question is wrong ! ;(

[tex]Corrected \\ Question...\\[/tex] (2^3)^2*(3^2)^3=6^5

the perimeter of square is 76 cm find are of square ​

Answers

Answer:

Given the information above, the area of the square is 361 cm²

Step-by-step explanation:

A square is a shape with four equal sides. So, in order to find the area of the square, we must find the length of each individual side. We can do this by dividing the perimeter by 4 because a square has 4 equal sides meaning they have the same lengths.

The perimeter of the square is 76. So, let's divide 76 by 4.

76 ÷ 4 = 19

So, the lengths of each sides in the square is 19cm.

In order to find the area, we must multiply the length and the width together. Since a square has equal sides, then we will multiply 19 by 19 to get the area.

19 × 19 = 361

So, the area of the square is 361 cm²

Answer:

361 cm^2

Step-by-step explanation:

The area of a square can be found by squaring the side length.

[tex]A=s^2[/tex]

A square has four equal sides. The perimeter is the sum of all four sides added together. Therefore, we can find one side length by dividing the perimeter by 4.

[tex]s=\frac{p}{4}[/tex]

The perimeter is 76 centimeters.

[tex]s=\frac{76 cm}{4}[/tex]

Divide 76 by 4.

[tex]s=19 cm[/tex]

The side length is 19 centimeters.

Now we know the side length and can plug it into the area formula.

[tex]A=s^2\\s=19cm[/tex]

[tex]A= (19 cm)^2[/tex]

Evaluate the exponent.

(19cm)^2= 19 cm* 19cm=361 cm^2

[tex]A= 361 cm^2[/tex]

The area of the square is 361 square centimeters.

1-Determine a solução dos sistemas abaixo pelo método de adição: a) {x + y = 5 {2x- y=9 b) {3x - y = 10 {x + y =18 Prfvr gente

Answers

a)

X + Y = 5

2X - Y = 9      

X + 2X + Y - Y = 5 + 9

3X = 14

X = 14/3

Para Y, basta substituir o valor de X em qualquer uma das 2 equacoes - arbitrariamente. Escolhendo a primeira:

X+ Y = 5

14/3 + Y = 5

Y = 5 - 14/3

Y = 1/3

.........................

b)

3X - Y = 10

X + Y = 18        

3X + X - Y + Y = 10 + 18

4X = 28

X = 7

Para Y, basta substituir o valor de X em qualquer uma das 2 equacoes - arbitrariamente. Escolhendo a segunda:

X + Y = 18

7 + Y = 18

Y = 18 - 7

Y = 11

Solve for x. 3x-91>-87 AND 17x-16>18

Answers

Answer & Step-by-step explanation:

For this problem, we have two inequalities to solve for x.

3x - 91 > -87

17x - 16 > 18

Now that we know what our inequalities are, we will solve them as if we are solving for the value of x.

3x - 91 > -87

Add 91 on both sides.

3x > 4

The solution for the first inequality is 3x > 4

Now let's do the second inequality.

17x - 16 > 18

Add 16 on both sides.

17x > 34

Divide by 17 on both sides.

x > 2

The soultion for the second inequality is x > 2

Answer:

The answer is x>2

Step-by-step explanation:

4x=24 solve equation

Answers

Answer:

x=6

Step-by-step explanation:

Rearrange:

Rearrange the equation by subtracting what is to the right of the equal sign from both sides of the equation :

                    4*x-(24)=0

Step by step solution :

STEP

1

:

Pulling out like terms

1.1     Pull out like factors :

  4x - 24  =   4 • (x - 6)

Equation at the end of step

1

:

STEP

2

:

Equations which are never true

2.1      Solve :    4   =  0

This equation has no solution.

A a non-zero constant never equals zero.

Solving a Single Variable Equation:

2.2      Solve  :    x-6 = 0

Add  6  to both sides of the equation :

                     x = 6

One solution was found :

x = 6

Answer:

x= 24/ 4

Step-by-step explanation:

You can simplify it

x= 6/1 which is x= 6

I need help factoring this question, Factor 4(20) + 84.

Answers

Answer:

164

Step-by-step explanation:

B for brackets

O for of

D for division

M for multiplication

A for addition

S for subtraction

You first start with the brackets (20) and multiply with 4 which is equal to 80 and then add it to 84 which makes 164

I hope this helps

I need help ASAP!!

Can someone explain this? And answer it? I am so confused!!

Answers

Answer:

Step-by-step explanation: hope this helps

If you make $3.80 an hour plus tips, what is your paycheck for the week if you worked 40 hours and made $250.00 dollars in tips?

Answers

Answer:

$402

Step-by-step explanation:

Hello!

If you made 3.80 an hour and worked 40 we can multiply these to find the total amount you earned.

3.80 * 40 = 152

You also made 250 in tips so we add that to the total

152+250 = 402

The answer is $402

Hope this helps!

I would make $402.00 by the end of the week

If x3 + ax2 – bx + 10 is divisible by x2 – 3x + 2,
find the values of
1) a-b
2) 2a-b

Answers

Answer: A=2 and B=13
Explanation: The Factor Theorem states that if a is the root of any polynomial p(x) that is if p(a)=0, then (x−a) is the factor of the polynomial p(x).

Let p(x)=x
3
+ax
2
−bx+10 and g(x)=x
2
−3x+2
Factorise g(x)=x
2
−3x+2:
x
2
−3x+2=x
2
−2x−x+2=x(x−2)−1(x−2)=(x−2)(x−1)
Therefore, g(x)=(x−2)(x−1)
It is given that p(x) is divisible by g(x), therefore, by factor theorem p(2)=0 and p(1)=0. Let us first find p(2) and p(1) as follows:
p(1)=1
3
+(a×1
2
)−(b×1)+10=1+(a×1)−b+10=a−b+11
p(2)=2
3
+(a×2
2
)−(b×2)+10=8+(a×4)−2b+10=4a−2b+18
Now equate p(2)=0 and p(1)=0 as shown below:
a−b+11=0
⇒a−b=−11.......(1)
4a−2b+18=0
⇒2(2a−b+9)=0
⇒2a−b+9=0
⇒2a−b=−9.......(2)
Now subtract equation 1 from equation 2:

(2a−a)+(−b+b)=(−9+11)
⇒a=2
Substitute a=2 in equation 1:
2−b=−11
⇒−b=−11−2
⇒−b=−13
⇒b=13
Hence, a=2 and b=13.

Evaluate the following expression. −8 × (−10) −7× 1/−1

Answers

Answer:

87

Step-by-step explanation:

[tex]-8\left(-10\right)-7 \times \frac{1}{-1}=87\\\\\mathrm{Apply\:rule}\:-\left(-a\right)=a\\\\=8\times \:10-7\times \frac{1}{-1}\\\\8\times \:10=80\\\\7\times \frac{1}{-1}=-7\\\\=80-\left(-7\right)\\\\\mathrm{Apply\:rule}\:-\left(-a\right)=a\\\\=80+7\\\\=87[/tex]

If sin Θ = 5 over 6, what are the values of cos Θ and tan Θ?

Answers

Answer:

Check explanation

Step-by-step explanation:

Sin∅=5/6

Opp=5. Hyp=6

Adj= (√6²+5²)

= √11

Cos∅=(√11)/6

Tan∅=5/(√11)

Answer: cos(theta) = sqrt(11)/6 ; tan(theta)=5/sqrt(11).

Explanation:
sin inverse is restricted to the first and fourth quadrant, so the other side of the triangle is positive sqrt(11) by the Pythagorean theorem.



A ship drops its anchor into the water and creates a circular ripple. The radius of the ripple increases at

a rate of 50 cm/s. If the origin is used as the location where the anchor was dropped into the water.

Find the equation for the circle 12 seconds after the anchor is dropped


Please write all the steps it’s for my summer school test and I need it done quick as possible thanks.

Answers

Answer:

The equation for the circle 12 seconds after the anchor is dropped is x^2 + y^2 = 360,000

Step-by-step explanation:

To find the equation for the circle 12 seconds when the radius of the ripple increases at a rate of 50 cm/s, the circle radius will be;

50 * 12 = 600 cm

Then place the equation inform of Pythagoras equation which is;

x^2 + y^2 = r^2

Where r is the radius

x^2 + y^2 = 600^2

x^2 + y^2 = 360,000

Then, the equation for the circle 12 seconds after the anchor is dropped is x^2 + y^2 = 360,000

Please answer this question now in two minutes

Answers

Answer:

m∠C = 102°

Step-by-step explanation:

This diagram is a Quadrilateral inscribed in a circle

The first step is to determine what m∠B

is

The sum of opposite angles in an inscribed quadrilateral is equal to 180°

m∠D + m∠B = 180°

m∠B = 180° - m∠D

m∠B = 180° - 80°

m∠B = 100°

Second step is we proceed to determine the exterior angles of the circle

m∠ADC = 2 × m∠B

m∠ADC = 2 × 100°

m∠ADC = 200°

m∠ADC = m∠CD + m∠AD

m∠AD = m∠ADC - m∠CD

m∠AD = 200° - 116°

m∠AD = 84°

The third step is to determine m∠BAD

m∠BAD = m∠AD + m∠AB

m∠BAD = 84° + 120°

m∠BAD = 204°

The final step Is to determine what m∠C is

It is important to note that:

m∠BAD is Opposite m∠C

Hence

m∠C = 1/2 × m∠BAD

m∠C = 1/2 × 204

m∠C = 102°

need help will give 5 stars.

Answers

Answer:

t=0.64

Step-by-step explanation:

h = -16t^2 +4t +4

We want h =0 since it is hitting the ground

0 = -16t^2 +4t +4

Using the quadratic formula

a = -16  b = 4  c=4

-b ± sqrt( b^2 -4ac)

----------------------------

         2a

-4 ± sqrt( 4^2 -4(-16)4)

----------------------------

         2(-16)

-4 ± sqrt( 16+ 256)

----------------------------

         -32

-4 ± sqrt( 272)

----------------------------

         -32

-4 ± sqrt( 16*17)

----------------------------

         -32

-4 ± sqrt( 16) sqrt(17)

----------------------------

         -32

-4 ± 4 sqrt(17)

----------------------------

         -32

Divide by -4

1 ±  sqrt(17)

----------------------------

         8

To the nearest hundredth

t=-0.39

t=0.64

Since time cannot be negative

t=0.64

Answer:

0.64  

Step-by-step explanation:

0 = -16t^2 + 4t + 4

-4(4t^2 - t -1) = 0

t = [-(-1) +/- sqrt (1 - 4*4*-1)] / 8)

t = 0.64, -0.39

answer is 0.64

The height of a building model is 2% of its actual height. If the building
model is 3 feet tall, how tall is the actual building?

Answers

Answer:

x = 150 feets

Step-by-step explanation:

Given that,

The height of a building model is 2% of its actual height.

The building model is 3 feet tall, h = 3 feet

We need to find the height of the actual building. Let it is x.

According to question,

h = 2% of x

We have, h = 3 feet

So,

[tex]x=\dfrac{h}{2\%}\\\\x=\dfrac{3}{2/100}\\\\x=150\ \text{feet}[/tex]

So, the actual height of the building is 150 feets.

Other Questions
At dinner, 100 students pass through the cafeteria line and were served meals. 40 fish entrees and 60 pasta entrees were served to the students. A total of 20 students chose neither entree. Assuming all students were served zero, one, or two entrees, how many students were served two entrees Which examination technique is the visualization of body parts in motion by projecting x-ray images on a luminous fluorescent screen? Which value would complete the last cell?(1 point)3.0100.025.04.0 The ratio of boys to girls in the ninth grade is 7 to 9. there are 218 girls set up a proportion to model this information TB MC Qu. 8-174 LBC Corporation makes and sells ... LBC Corporation makes and sells a product called Product WZ. Each unit of Product WZ requires 2.0 hours of direct labor at the rate of $16.00 per direct labor-hour. Management would like you to prepare a Direct Labor Budget for June. The company plans to sell 39,000 units of Product WZ in June. The finished goods inventories on June 1 and June 30 are budgeted to be 610 and 110 units, respectively. Budgeted direct labor costs for June would be: Consider the circle of radius 10 centered at the origin. Find an equation of the line tangent to the circle at the point (6, 8) Menlo Company distributes a single product. The companys sales and expenses for last month follow: Total Per unitSales $314,000 $20Variable expenses 219,800 14Contribution margin 94,200 6 Fixed expenses 75,000Net operating income 19,200Required: a. What is the monthly break-even point in unit sales and in dollar sales? b. Without resorting to computations, what is the total contribution margin at the break-even point? c. How many units would have to be sold each month to attain a target profit of S27,600? d. Verify your answer by preparing a contribution format income statement at the target sales level. e. Refer to the original data. Compute the company's margin of safety in both dollar and percentage terms. f. What is the company's CM ratio? If sales increase by $76,000 per month and there is no change in fixed expenses, by how much would you expect monthly net operating income to increase? In a(n) __________, the subject comes before the verb.(1 point) 1 singular noun 2 standard sentence 3 inverted sentence 4 prepositional phrase An aluminum rod 17.400 cm long at 20C is heated to 100C. What is its new length? Aluminum has a linear expansion coefficient of 25 10-6 C-1. Solve for x: x/25 > 5 Use the map as a reference and select the best answer. In ____ the cataumbo lightning storms last for about 10 hours! La hermana de Sofa es ________. A) Sofa El padre Leonor es ________. B) Letizia La madre Leonor y Sofa es ________. C) Leonor La abuela de Leonor y Sofa es ________. D) Felipe IV Unprecedented industrial growth in the country of Remo has created serious environmental problems because factories there lack adequate pollution control systems. Remo is developing a clean growth plan that includes environmental regulations that will require the installation of such systems. Since no companies in Remo currently produce pollution-control systems, the plan, if implemented, will create significant opportunities for foreign exporters to market pollution-control systemsWhich of the following, if true, most seriously weakens the argument?A. The clean growth plan will provide tax incentives for local business to develop and manufacture pollution-control systemsB. Foreign exporters would provide factory-trained technicians to maintain the pollution-control systems sold in Remo.C. Industrial lobbyists sponsored by local business in Remo are trying to prevent the implementation of the government regulationsD. The regulations that Remo plans to implement are much less strict than those in neighboring nations.E. Pollution in Remo has caused serious health problems for workers, contributing to a significant increase in the number of workdays lost to illness When you encounter a T-critical value of 2.50, which T-obtained value would reject the null hypothesis? A) Tobt = 2.75 B) Tobt = -2.75 C) Tobt = 2.40 D) Tobt = - 2.40 Wyckam Manufacturing Inc. has provided the following information concerning its manufacturing costs: Fixed Cost per Month Cost per Machine-Hour Direct materials $ 5.40 Direct labor $ 42,400 Supplies $ 0.30 Utilities $ 1,700 $ 0.25 Depreciation $ 15,200 Insurance $ 11,600 For example, utilities should be $1,700 per month plus $0.25 per machine-hour. The company expects to work 4,200 machine-hours in June. Note that the companys direct labor is a fixed cost.Required:Prepare the company's planning budget for manufacturing costs for June. Water discharging into a 10-m-wide rectangular horizontal channel from a sluice gate is observed to have undergone a hydraulic jump. The flow depth and velocity before the jump are 0.8m and 7m/s, respectively. Determine (a) the flow depth and the Froude number after the jump (b) the head loss (c) the dissipation ratio. Gavin goes to the market and buys one rectangle shaped board. The length of the board is 16 cm and width of board is 10 cm. If he wants to add a 2 cm wooden border around the board, what will be the area of the rectangle board? Analyze the following scenarios to determine who can appropriately access health information.1. Mrs. John Smith is requesting the emergency room records from last week of her daughter, Katy. Mrs. Smith is the noncustodial parent of Katy, who lives with her dad. Should you release the records to her? Why or why not?2. Mr. Fred Mitchell is requesting the birth record for Amy, his birth daughter. Mr. and Mrs. Mitchell gave Amy up for adoption four years ago. Should you release the records to him? Why or why not?3. Mrs. Lynn Olsen is requesting the lab results of her husband, Tim. She has a note. signed by him, giving his permission for her to have the records. Should you release the records to her? Why or Why not?4. An investigator from the Health and Human Services department is conducting an audit of patient records and has provided a list of records that they want to review. Should you release the information to the investigator? Why or why not?5. Dr. Rex Harrisson is requesting the medical records of Martha Flynn. He states he is a family friend and has been asked by Mrs. Flynn's son to review her last inpatient admission for appropriateness of care. Should you release the records to Dr. Harrison? Why or why not? The width of a rectangle measures (8.3c-8.4d)(8.3c8.4d) centimeters, and its length measures (5.3c+4.8d)(5.3c+4.8d) centimeters. Which expression represents the perimeter, in centimeters, of the rectangle? The energy change in an endothermic reaction is: A. Internal B. External C. Negative D. Positive